【考研类试卷】工商管理硕士综合知识-3及答案解析.doc

上传人:sofeeling205 文档编号:1383766 上传时间:2019-12-03 格式:DOC 页数:13 大小:101KB
下载 相关 举报
【考研类试卷】工商管理硕士综合知识-3及答案解析.doc_第1页
第1页 / 共13页
【考研类试卷】工商管理硕士综合知识-3及答案解析.doc_第2页
第2页 / 共13页
【考研类试卷】工商管理硕士综合知识-3及答案解析.doc_第3页
第3页 / 共13页
【考研类试卷】工商管理硕士综合知识-3及答案解析.doc_第4页
第4页 / 共13页
【考研类试卷】工商管理硕士综合知识-3及答案解析.doc_第5页
第5页 / 共13页
点击查看更多>>
资源描述

1、工商管理硕士综合知识-3 及答案解析(总分:115.00,做题时间:90 分钟)一、B单项选择题/B(总题数:1,分数:115.00)Now that youve gotten a little more confident, were going to give you the opportunity to practice “without the training wheels.“ When you want to check your work, you can turn to the solutions at the end. If you arent familiar with s

2、ome of the mathematical concepts, make a note to pay particular attention to that chapter in this book; this practice set covers a wide range of topics tested on the GMAT.On all data sufficiency problems, the answer choices are the same (as youve learned). Weve put them here for your reference. Stat

3、ement (1) ALONE is sufficient, but statement (2) alone is not sufficient. Statement (2) ALONE is sufficient, but statement (1) alone is not sufficient. BOTH statements TOGETHER are sufficient, but NEITHER statement ALONE is sufficient. EACH statement ALONE is sufficient. Statements (1) and (2) TOGET

4、HER are NOT sufficient.(分数:115.00)(1).Is x an integer? (1) x is evenly divisible by (分数:5.00)A.B.C.D.(2).What is the ratio of 2a to b? (1) (分数:5.00)A.B.C.D.(3).What is the average of a list of n consecutive integers? (1) The smallest number in the list is 5. (2) n=8(分数:5.00)A.B.C.D.(4).Is s=r?(1) s2

5、=r2(2) s is positive.(分数:5.00)A.B.C.D.(5).In triangle ABC, what is the length of AB? (1) The length of BC is 5 and the length of AC is 12. (2) Angle C=90.(分数:5.00)A.B.C.D.(6).If there are only red, blue, and green marbles in a jar, what is the ratio of red to blue marbles? (1) The ratio of red to gr

6、een marbles is 2:3. (2) The ratio of green to blue marbles is 6:5.(分数:5.00)A.B.C.D.(7).Over a holiday weekend, a certain car dealer sold off 4 of the cars on its lot. If the cars sold for an average of $6,000 each, how many cars were on the dealers lot at the beginning of the weekend? (1) The averag

7、e value of the remaining cars on the lot is $5,000. (2) The car dealer made $48,000 in car sales over the weekend.(分数:5.00)A.B.C.D.(8).What is the value of ? (1) (分数:5.00)A.B.C.D.(9).Was the price of a certain stereo in March greater than its price in May? (1) The price of the stereo in March was 80

8、 percent of its price in April. (2) The price of the stereo in April was 120 percent of its price in May.(分数:5.00)A.B.C.D.(10).If the square root of t is a real number, is the square root of t positive?(1) t0(2) t20(分数:5.00)A.B.C.D.(11).If the ratio of brown cars to blue cars in a certain parking lo

9、t is 2:3, how many brown cars are in the lot? (1) There are 15 blue cars in the lot. (2) There are 25 cars total in the lot.(分数:5.00)A.B.C.D.(12).What is the value of f2-g2?(1) f=-g+8(2) f=g-2(分数:5.00)A.B.C.D.(13).A shelf contains only books of poetry, short stories, and non-fiction. If Jana draws a

10、 book randomly off the shelf, what is the probability that the book will be non-fiction? (1) There are 15 books on the shelf. (2) There are 4 books of poetry and 5 books of short stories on the shelf.(分数:5.00)A.B.C.D.(14).A job opening was posted in September and again in January. In September, the

11、number of applicants for the position was 60. What was the percent change in the number of applicants from September to January? (1) The number of applicants in January was one-third the number of applicants in September. (2) The number of applicants in January was 20.(分数:5.00)A.B.C.D.(15).Is a320?(

12、1) a480(2) a5200(分数:5.00)A.B.C.D.(16).What is the length of a side of a certain cube? (1) The volume of the cube is 27. (2) The surface area of the cube is 54.(分数:5.00)A.B.C.D.(17).How many prime factors of x are also prime factors of y? (1) x=30 (2) y is a multiple of x.(分数:5.00)A.B.C.D.(18).Is m0?

13、(1) (分数:5.00)A.B.C.D.(19).What is the value of ? (1) (分数:5.00)A.B.C.D.(20).There are twenty students in a class, all of whom scored between 0 and 100 on their final exams. If the class average was 85, how many people scored below the average score on the exam? (1) Twelve people in the class scored h

14、igher than an 85. (2) The lowest eight scores added up to 640.(分数:5.00)A.B.C.D.(21).Advertisement:Of the many over-the-counter medications marketed for the relief of sinus headache, SineEase costs the least per dose. And SineEase is as effective per dose as the most effective of those other medicati

15、ons. So for relief from sinus headaches, SineEase is the best buy.Which of the following, if true, most seriously weakens the argument above? A. Most of the over-the-counter medications marketed for the relief of sinus headache are equally effective per dose in providing such relief. B. Many of the

16、over-the-counter medications marketed for the relief of sinus headache contain the same active ingredient as SineEase. C. People who suffer from frequent sinus headaches are strongly advised to consult a doctor before taking any over-the-counter medication. D. An over-the-counter medication that is

17、marketed for the relief of symptoms of head cold is identical in composition to SineEase but costs less per dose. E. The per dose price for any given over-the-counter medication marketed for the relief of sinus headache is higher for smaller packages than it is for larger packages.(分数:5.00)A.B.C.D.(

18、22).Crimes are mainly committed by the young, and for this reason merely increasing the number of police officers or expenditures on police services has little effect on reducing the crime rate. In fact, the only factor associated with a crimerate drop is a decrease in the number of People in the co

19、mmunity aged fourteen to thirty.The findings above can best serve as part of an argument against A. the likelihood that any law enforcement program will be effective in reducing the crime rate within a short time. B. increasing prison terms for young people found guilty of crimes. C. introducing com

20、pulsory military conscription for people aged seventeen to nineteen. D. raising the age at which students are permitted to leave school. E. a communitys plan to increase the number of recreational and educational activities in which young adults can participate.(分数:5.00)A.B.C.D.(23).A member of the

21、British Parliament is reputed to have said, “The First purpose of good social reform is to increase the sum total of human happiness. So, any reform which makes somebody happy is achieving its purpose. Since the reform I propose would make my constituents happy, it is a good social reform. “Which on

22、e of the following, if true, most seriously weakens the argument attributed to the member of Parliament? A. Different things make different people happy. B. The proposed reform would make a few people happy, but would not increase the happiness of most other people. C. The proposed reform would affe

23、ct only the member of Parliaments constituents and would make them happy. D. Increasing some peoples happiness might not increase the sum total of human happiness if others are made unhappy. E. Good social reforms usually have widespread support.(分数:5.00)A.B.C.D.工商管理硕士综合知识-3 答案解析(总分:115.00,做题时间:90 分

24、钟)一、B单项选择题/B(总题数:1,分数:115.00)Now that youve gotten a little more confident, were going to give you the opportunity to practice “without the training wheels.“ When you want to check your work, you can turn to the solutions at the end. If you arent familiar with some of the mathematical concepts, make

25、 a note to pay particular attention to that chapter in this book; this practice set covers a wide range of topics tested on the GMAT.On all data sufficiency problems, the answer choices are the same (as youve learned). Weve put them here for your reference. Statement (1) ALONE is sufficient, but sta

26、tement (2) alone is not sufficient. Statement (2) ALONE is sufficient, but statement (1) alone is not sufficient. BOTH statements TOGETHER are sufficient, but NEITHER statement ALONE is sufficient. EACH statement ALONE is sufficient. Statements (1) and (2) TOGETHER are NOT sufficient.(分数:115.00)(1).

27、Is x an integer? (1) x is evenly divisible by (分数:5.00)A.B. C.D.解析:A reminder: The question stem starts with is, so this is a yes or no problem. The first step is to determine whether statement (1) is sufficient, which we can do by trying out some real numbers. If x is 2, which is evenly divisible b

28、y *, then the answer would be yes. But if x were 1.5, which is also evenly divisible by *, then the answer would be no. Since you can get either yes or no, statement (1) is not sufficient and you should cross off A and D. Now lets move on to statement (2). There are no non-integer numbers that are d

29、ivisible by 2, so yes is the only possibility. Statement (2) is sufficient, so we can eliminate C and E, and the correct answer is B.(2).What is the ratio of 2a to b? (1) (分数:5.00)A. B.C.D.解析:Statement (1) says that * We are looking for the ratio *. If you substitute 2 for * in that ratio, you get 2

30、 times 2, which equals 4. Statement (1) is sufficient. Cross off B, C, and E. How about statement (2)? Remember, you have to forget about the information you got from statement (1). Since there are several values for a and b that would add up to 5, lets try a couple of variations to see if we always

31、 get the same ratio. If a is 3 and b is 2, the ratio of 2a to b would be 6:2 or 3:1. But if we used a=2 and b=3 instead (and nothing tells us that we cant), the answer to the question stem becomes 4:3. When we can get different answers, we can tell the statement is not sufficient, so we can eliminat

32、e D. A is the correct answer.(3).What is the average of a list of n consecutive integers? (1) The smallest number in the list is 5. (2) n=8(分数:5.00)A.B.C. D.解析:Statement (1) tells us the smallest number, but not how many numbers are in the list. It is not sufficient. Cross off A and D.Statement (2)

33、tells us the value of n, so the question becomes: What is the average of a list of 8 consecutive integers? Since those integers can be large, small, or even negative, we have no way to tell what the average is. Statement (2) is not sufficient, so we can eliminate B.Now lets put the two statements to

34、gether. If we know that 5 is the smallest number in a list of 8 consecutive numbers, then we can easily reconstruct the list and find the average. So the statements are sufficient when put together, and the answer is C.(4).Is s=r?(1) s2=r2(2) s is positive.(分数:5.00)A.B.C.D.解析:Heres another question

35、that is looking for a yes or no answer. Statement (1) allows for both positive and negative values of both s and r, so we cannot tell ff s = r. Statement (1) is not sufficient; cross off A and D. Statement (2) just tells us that s is a positive number; we dont know anything about r at all and cannot

36、 tell if they are equal; statement (2) alone is not sufficient. Lets eliminate B. What if we combine the two? Statement (2) tells us that s is positive, but r could still be either positive or negative. We still cannot tell if they are equal, so the answer is E.(5).In triangle ABC, what is the lengt

37、h of AB? (1) The length of BC is 5 and the length of AC is 12. (2) Angle C=90.(分数:5.00)A.B.C. D.解析:Statement (1) alone is tempting if you are familiar with the right triangles that tend to be tested over and over on the GMAT; however, we dont know from statement (1) that ABC is a right triangle, so

38、we should cross off A and D. With only statement (2), we certainly dont have enough information to answer the question, so B can be eliminated. Together, though, we can complete the picture of a right triangle with two known sides, which means we can figure out the third side using Pythagorean Theor

39、em, and our answer is C. (Since the squares of the two legs add up to 169, the hypotenuse is 13.)(6).If there are only red, blue, and green marbles in a jar, what is the ratio of red to blue marbles? (1) The ratio of red to green marbles is 2:3. (2) The ratio of green to blue marbles is 6:5.(分数:5.00

40、)A.B.C. D.解析:Statement (1) doesnt give us any information about the number of blue marbles at all. It is not sufficient and we can cross off A and D. Statement (2) doesnt give us any information about the number of red marbles, so we can eliminate B. When we combine both statements, we have ratios t

41、hat involve all three colors of marbles. Lets see what happens when we use some real numbers. If there are 6 green marbles, then there are 4 red marbles and 5 blue marbles, making the ratio of red to blue marbles 4:5. If we used a different number for the green marbles-say, 12-we would end up with t

42、he same ratio of red to blue. This happens frequently with relative numbers such as ratios, proportions, averages, and probability; keep in mind that you dont always have to know the actual numbers to know the ratios. Statements (1) and (2) together are sufficient, so the answer is C.(7).Over a holi

43、day weekend, a certain car dealer sold off 4 of the cars on its lot. If the cars sold for an average of $6,000 each, how many cars were on the dealers lot at the beginning of the weekend? (1) The average value of the remaining cars on the lot is $5,000. (2) The car dealer made $48,000 in car sales o

44、ver the weekend.(分数:5.00)A.B. C.D.解析:Given statement (1) alone, we only know average values of both the cars that were sold and the cars that remained; the dealer could have sold 4 cars and had 1 left, or sold 8 cars and had 2 left, or an infinite number of other options. So we can eliminate A and D

45、. With statement (2), we are able to find the number of cars sold by dividing the total sales by the average price. 48,000/6,000=8, so the dealer sold 8 cars. Since that is * of the cars on the lot, the dealer started off with 10 cars. Statement (2) is sufficient, so the answer is B.(8).What is the

46、value of ? (1) (分数:5.00)A.B.C.D.解析:Statement (1) doesnt tell us about g at all, so it is insufficient, and we can cross off A and D. Similarly, statement (2) doesnt tell us about f, so we can eliminate B. If we put the statements together, we have only two equations for our three variables, so we st

47、ill cannot solve. Since the statements together are not sufficient, the answer is E.(9).Was the price of a certain stereo in March greater than its price in May? (1) The price of the stereo in March was 80 percent of its price in April. (2) The price of the stereo in April was 120 percent of its pri

48、ce in May.(分数:5.00)A.B.C. D.解析:Again, this problem is looking for a yes or no answer. Statement (1) doesnt tell us anything about the May price, so it is not sufficient. We can cross off A and D. Statement (2) doesnt tell us anything about March, so it is not sufficient and we can cross off B. With

49、both statements together, we know relative prices, and no matter what numbers we were to assume, the May price ends up slightly higher than the March price. So the answer is definitely no, meaning that both statements together are sufficient, and the answer is C.(10).If the square root of t is a real number, is the square root of t positive?(1) t0(2) t20(分数:5.00)A.B.C.D.解析:This question is looking for a definite yes or no answer. Statement (1) is not sufficient since

展开阅读全文
相关资源
猜你喜欢
相关搜索

当前位置:首页 > 考试资料 > 大学考试

copyright@ 2008-2019 麦多课文库(www.mydoc123.com)网站版权所有
备案/许可证编号:苏ICP备17064731号-1